Q17

 
AmandaG135
Thanks Received: 0
Vinny Gambini
Vinny Gambini
 
Posts: 2
Joined: July 15th, 2018
 
 
 

Q17

by AmandaG135 Fri Jan 18, 2019 12:37 pm

Can someone please provide an answer to this question, and possibly tips on how to handle these "substitute" questions for games.
I picked E and had eliminated A.
User avatar
 
ohthatpatrick
Thanks Received: 3807
Atticus Finch
Atticus Finch
 
Posts: 4661
Joined: April 01st, 2011
 
This post thanked 1 time.
 
 

Re: Q17

by ohthatpatrick Mon Jan 21, 2019 1:46 am

The correct answer has to result in an identical game.

Answers are wrong because they're Too Restrictive (they forbid something that used to be allowed) or Too Permissive (they allow something that used to be forbidden).

As you're subjecting each answer choice to that 2-part analysis, you should ask yourself these questions:

1. Too Restrictive: as you read each rule, ask ...
- Was this true before? (if not, eliminate)
- Do I have any counterexamples to this rule? (if so, eliminate)

2. Too Permissive: as you read each rule, ask ...
- Does this "do the work" of the original rule (if so, pick it)
- Can I follow this new rule, but break the original rule? (if so, eliminate it)


In terms of where correct answers come from, about 50% of them are written with the same strategy:
- We're replacing a rule about A and B
- One (or both) of those characters is in some OTHER rule with Z
- The correct answer re-writes the original rule in a way that swaps out the dude who was in both rules and inserts Z instead.


For example, if you have these rules
F - J
JM

And they say, "replace F - J",
you'd think, "J is also in the JM rule. I should look for an answer that connects F and M."

possible correct answer:
A) F - M


For this one, the rule we're replacing involves R and M. Do any other rules involve R or M?
Yes, rule 1 involves N and R. (so an answer might connect N and M)
Yes, rule 4 involves L and R. (so an answer might connect L and M)

Do we see any answers connecting N and M or L and M?
Yes, (A) and (E).

We need our new rule to "do the work" of R --> M.

(A) If L is out, M is in. Since rule 4 says "if R is in, L is out", we would now have
R --> ~L --> M

So (A) seems correct. It gives us the same reality we had with rule 3, that when R is in, M is in.

(E) If M is in, N is out.
We knew from rule 1 that if N is out, R is in. So we would now have
M --> ~N --> R

But that's not the original rule. The original rule was R -> M, not M -> R.
We can eliminate this mismatch.

Knowing that tendency to "swap out the character who's in other rules with their rule-buddy from some other rule" put us on the fast track to seeing that (A) is correct.

Otherwise, we'd have to eliminate them by saying they violate previous possibilities or allow us to break the original rule.


(B) This was true before. If we chained together a couple of our rules, we knew that
~L -> ~N -> R
But .. this doesn't force M to be in.

We could follow this new rule and break the original rule:
R | L M N P Q

(C) This was true before. If we chained together a couple rules, we knew that
~R --> N --> L
But .. this doesn't force M to be in.

We could follow this new rule and break the original rule:
R | L M N P Q

(D) Before, we knew this about L being open:
L -> ~R -> N

L being open didn't tell us anything about M. This appears to be too restrictive.
Previously, we were allowed to do this:
L M N | R P Q

This rule would no longer allow that, so it can't be describing an identical game.

(E) M didn't trigger anything before, so this isn't going to match our original game.


Hope this helps.
 
HannahS74
Thanks Received: 0
Vinny Gambini
Vinny Gambini
 
Posts: 5
Joined: June 16th, 2017
 
 
 

Re: Q17

by HannahS74 Sun Mar 17, 2019 9:09 am

I don't totally get how C breaks old rules.

I get
(C) This was true before. If we chained together a couple rules, we knew that
~R --> N --> L

But why do we need M to be in? I thought that before, R was the outcome when it was out, so if it was out, M (the trigger) could be either in or out. I don't see how following this new rule and getting : Out: R | In: L M N P Q breaks old rules. That's the set up I had for Question 13 when only one element could be out.

Any help is much appreciated, thanks!
User avatar
 
ohthatpatrick
Thanks Received: 3807
Atticus Finch
Atticus Finch
 
Posts: 4661
Joined: April 01st, 2011
 
 
 

Re: Q17

by ohthatpatrick Thu Mar 21, 2019 6:33 pm

It looks like you just misjudged how I was writing scenarios.

I always have IN on the left. I actually don't know if I've ever seen anyone every put OUT on the left, so I didn't bother to label it. :)

You thought I was writing this:
Out: R | In: L M N P Q

I was writing this:
In: R | Out: L M N P Q

Any scenario I wrote in that thread was intended to show IN on the left and OUT on the right.

Do you see how this scenario
In: R | Out: L M N P Q
is following the rule for (C) but breaking the original rule of "R --> M" that it's supposed to be replacing?